f(x) = x^2. What is g(x)?

F(x) = X^2. What Is G(x)?

Answers

Answer 1

The point shown on g(x) is (1,3)

If x = 1 and y = 3, this means x is multiplied by 3 ( 1 x 3 = 3)

This makes g(x) 3x^2

The answer is A.

F(x) = X^2. What Is G(x)?

Related Questions

solve 2(1/9)× = 2/81 for x​

Answers

Answer: x=1/9

Step-by-step explanation:

[tex]2\left(\frac{1}{9}\right)x=\frac{2}{81}[/tex]

[tex]\frac{2}{9}x=\frac{2}{81}[/tex]

multiply both sides by 9

[tex]9\cdot \frac{2}{9}x=\frac{2\cdot \:9}{81}[/tex]

[tex]2x=\frac{2}{9}[/tex]

divide 2 on both sides

[tex]x=\frac{1}{9}[/tex]

need help please. Will give you 5-stars and a big thank you comrades

Answers

Answer:

first answer

Step-by-step explanation:

(8x³ - 22x² - 4) / (4x - 3)

when you do long division you get the first answer

Carey earns $9.75 working part time on weekends. The table below shows the amount, a, Carey earns for working h hours. Carey’s Earnings h 0 1 3 a $0 $9.75 ? Which value completes the table to show the amount Carey earns for working 3 hours?

Answers

Answer:

$29.25

Step-by-step explanation:

For every 1 hour, Carey earns $9.75.  Multiply $9.75 by 3 to find out how much she earns for 3 hours of work.

$9.75 × 3 = $29.25

Carey earns $29.25 for working 3 hours.

Answer:

29.25

Step-by-step explanation:

I got it right on edge!! trust me

What is this used for and how do i use it..?

Answers

you have to solve each one to get your answer and I think that your answer will be inside the circle

Answer:

This is called the Unit Circle. It is used in trigonometry. It had a radius of 1.

It helps you when using the trig function of sin cos and tan.

Hope this helps!!!!

Step-by-step explanation:

What is the area of this composite figure?

Answers

Answer:

C.) 7.14 in²

Step-by-step explanation:

The figure is made up of a square and a circle. The circle is divided in half and each piece is set on one side of the square. This means that the diameter of the circle is equal to the length of the sides of the square, 2 inches.

The area of the square can be found by multiplying length times the width:

[tex]2*2=4[/tex]

The area of the square is 4 inches, and since we multiplied two lengths, we square the value:

A=4in²

Now find the area of the circle using the formula:

[tex]A=\pi r^2[/tex]

The radius is half of the diameter, so the radius is 1. Insert values and solve:

[tex]A=\pi *1^2\\\\A=\pi *1\\\\A=\pi[/tex]

The area of the circle is equal to π. Add the values together:

[tex]4+\pi =7.14[/tex]

The area of the figure is 7.14 in²

:Done

An octagonal pyramid ... how many faces are there, how many vertices and how many edges? A triangular prism ... how many faces are there, how many vertices and how many edges? a triangular pyramid ... how many faces are there, how many vertices and how many edges?

Answers

1: 8 faces and 9 with the base 9 vertices and 16 edges

2: 3 faces and 5 with the bases 6 vertices and 9 edges

3: 3 faces and 4 with the base 4 vertices and 6 edges

1: 8 faces and 9 with the base 9 vertices and 16 edges

2: 3 faces and 5 with the bases 6 vertices and 9 edges

3: 3 faces and 4 with the base 4 vertices and 6 edges

Which expressions are equivalent to 2(b+3c)2(b+3c)2, left parenthesis, b, plus, 3, c, right parenthesis ?

Choose all answers that apply:

Choose all answers that apply:


(Choice A)

A

3(b+2c)3(b+2c)3, left parenthesis, b, plus, 2, c, right parenthesis


(Choice B)

B

(b+3c)+(b+3c)(b+3c)+(b+3c)left parenthesis, b, plus, 3, c, right parenthesis, plus, left parenthesis, b, plus, 3, c, right parenthesis


(Choice C)

C

2(b)+2(3c)2(b)+2(3c)2,

Answers

Answer:

B. (b+3c)+(b+3c) C. 2(b)+2(3c)

Step-by-step explanation:

Given this expression 2(b+3c), its equivalent expression is derived by simply opening up the bracket as shown below;

Open the parenthesis by multiplying the constant outside the bracket with all the variables in parenthesis.

= 2(b+3c)

=  2(b)+ 2(3c)

= 2b +2*3*c

= 2b +6c

It can also be written as sum of b+3c in 2 places i.e (b+3c)+(b+3c) because multiplying the function b+3c by 2 means we are to add the function by itself in two places.

Hence the equivalent expression are (b+3c)+(b+3c) and 2(b)+2(3c) or 2b+6c

The Muller family are on holiday in New Zealand. a. They change some euros (€) and receive $1962 (New Zealand dollars). The exchange rate is €1 = $1.635. Calculate the number of euros they change. [3] b. The family spend 15% of their New Zealand dollars on a tour. Calculate the number of dollars they have left. [4]

Answers

Answer:

a. €1200;$1667.70

Step-by-step explanation:

a. Number of euros

[tex]\text{euros} = \$1962 \times \dfrac{\text{1 euro}}{\text{\$1.635}} = \textbf{1200 euros}[/tex]

b. Dollars remaining

Dollars on hand                        = $1962.00

Less 15 % spent = 0.15 × 1962 =   -294.30

Balance remaining                   =  $1667.70

Find f(x) and g(x) so the function can be expressed as y = f(g(x)). (1 point) [tex]y=\frac{7}{x^{2} } +10[/tex]

Answers

Answer:

The functions are [tex]f(x) = 7\cdot x+10[/tex] and [tex]g(x) = \frac{1}{x^{2}}[/tex], respectively.

Step-by-step explanation:

Let suppose that [tex]g(x) = \frac{1}{x^{2}}[/tex], then [tex]f(g(x))[/tex] is:

[tex]f(g(x)) = 7\cdot \left(\frac{1}{x^{2}} \right) + 10[/tex]

[tex]f(g(x)) = 7\cdot g(x) + 10[/tex]

Thus,

[tex]f(x) = 7\cdot x + 10[/tex]

The functions are [tex]f(x) = 7\cdot x+10[/tex] and [tex]g(x) = \frac{1}{x^{2}}[/tex], respectively.

What expression has the same value as -3/2-(2-3/8)+3/2

Answers

Answer:

[tex]\dfrac{-3}{2}-(2-\dfrac{3}{8})+\dfrac{3}{2}=\dfrac{-13}{8}[/tex]

Step-by-step explanation:

We need to find the value of expression [tex]\dfrac{-3}{2}-(2-\dfrac{3}{8})+\dfrac{3}{2}[/tex].

Firstly solving the second term as :

[tex](2-\dfrac{3}{8})=\dfrac{16-3}{8}=\dfrac{13}{8}[/tex]

Now the above expression becomes,

[tex]\dfrac{-3}{2}-(2-\dfrac{3}{8})+\dfrac{3}{2}\\=\dfrac{-3}{2}-\dfrac{13}{8}+\dfrac{3}{2}[/tex]

-3/2 and +3/2 equals 0.

It means that, [tex]\dfrac{-3}{2}-(2-\dfrac{3}{8})+\dfrac{3}{2}=\dfrac{-13}{8}[/tex]

For each of the following paralellogram calculate the unknown angles marked. x, y and z

Answers

Answer:

x = 50°, y = z = 40°

Step-by-step explanation:

x = 50° ( Alternate angle )

z = 180° - (110 + 30)° = 180° - 140° = 40° ( sum of angles in Δ )

y = z = 40° ( Alternate angles )

HELP ASAP ITS SO HARD! Kelsey did the following division problem. Her teacher says that the quotient she found is wrong. −2 5/6 ÷ 1 1/3 −17/6 ÷ 4/3 −6/17• 3/4 −6×3 divided by 17×4 −18/68 −9/34 A. Identify what Kelsey did wrong in her calculations. B. Find the correct quotient, showing all of your calculations.

Answers

Part A

Her steps were

[tex]-2 \frac{5}{6} \div 1 \frac{1}{3}\\\\-\frac{17}{6} \div \frac{4}{3}\\\\-\frac{6}{17} \times \frac{3}{4}\\\\-\frac{6\times 3}{17\times4}\\\\-\frac{18}{68}\\\\-\frac{9}{34}\\\\[/tex]

Kelsey made a mistake on line 3. Note how the 17/6 flips to 6/17. This is not correct. You keep the first fraction the same, but you do flip the second fraction. This only applies when you divide two fractions.

The third step should look like [tex]-\frac{17}{6}\times \frac{3}{4}[/tex]

=======================================================

Part B

Here's what she should have written

[tex]-2 \frac{5}{6} \div 1 \frac{1}{3}\\\\-\frac{17}{6} \div \frac{4}{3}\\\\-\frac{17}{6} \times \frac{3}{4}\\\\-\frac{17\times 3}{6\times 4}\\\\-\frac{51}{24}\\\\-\frac{17}{8}\\\\[/tex]

If you want to convert that improper fraction to a mixed number, then you could do something like this

[tex]-\frac{17}{8} = -\frac{16+1}{8}\\\\-\frac{17}{8} = -\frac{16}{8}-\frac{1}{8}\\\\-\frac{17}{8} = -2 \frac{1}{8}\\\\[/tex]

Or you could divide 17 over 8 using long division to get 2 remainder 1. The 2 is the quotient that goes to the left of the 1/8. The remainder of 1 is the numerator of 1/8.

Which data set matches the box-and-whisker plot?
A) 12 13 15 19 23 23 25 26.5 28 30
B) 15 13 19 21 23 24 27 29 32
C) 11 31 13 15 19 21 21 25 27 29 31
D) 11 13 15 19 23 23 24 26.5 28 33​

Answers

Answer:

D) 11 13 15 19 23 23 24 26.5 28 33​

Step-by-step explanation:

The box-and-whisker plot displayed above has the following key values that we can use to identify which of the given data set it matches. It has:

Minimum value = 11

Q1 = 15

Median = 23

Q3 = 26

Maximum value = 33

From the options given, using just the max and min value, we can conclude that the data set in option D matches the box plot.

The data set in option D has a minimum value of 11, and a maximum value of 33.

Wait times at a dentist's office are typically 21 minutes, with a standard deviation of 2 minutes. What percentage of people should be seen by the doctor between 17 and 25 minutes for this to be considered a normal distribution?

Answers

ANSWER: 95%

HOW:
95% of a group of data with a normal distribution is between two standard deviations to the right and left

Answer:

95%

Step by step explanation:

z = 17-21 / 2 and z = 25-21/2

z=-2 (2.28%) z=2 (97.72%)

97.72 - 2.28 = 5.44

100% - 5.44% is about equal to 95%

stagg high school has a rectangular swiming pool the area of the water in the pool is 1,800 meters squared the length is twice the width what is the perimeter of the pool find the length and width. SHOW WORK

Answers

Step-by-step explanation:

L*b=1800m^2

L=2b

2b*b=1800

2b^2=1800

b^2=900

b=30m

L=2*30

=60m

Perimeter=2(l+b)

=2(60+30)

=2*90

=180m

Ten people were chosen at random and surveyed. The survey asked participants for the number of hours they sleep per night and the amount of their annual income. Letting X represent the number of hours the participant sleeps per night and Y represent the participant's annual income, the surveyor calculated the correlation coefficient between X and Y to be 0.29. Interpret the correlation coefficient calculated by choosing the statement below which correctly describes the correlation between X and Y. A. weak negative correlation B. strong negative correlation C. strong positive correlation D. weak positive correlation

Answers

Answer:

A. R=0.86; strong correlation

Step-by-step explanation:

The correlation coefficient of 0.29 indicates that the correlation is a weak positive correlation. Thus option (D) is the correct answer.

What is correlation?

"Correlation is a statistical tool that studies the relationship between two variables. Data sets have a positive correlation when they increase together, and a negative correlation when one set increases as the other decreases".

For the given situation,

Correlation coefficient = 0.29

Positive correlation: the two variables change in the same direction.

Negative correlation: the two variables change in opposite directions.

No correlation: there is no association or relevant relationship between the two variables.

The correlation coefficient lies between 0 to 0.3 indicating that the correlation is a weak positive correlation.

Hence we can conclude that option (D) weak positive correlation is the correct answer.

Learn more about correlation here

https://brainly.com/question/10721912

#SPJ2

Show all work to solve the equation for x. If a solution is extraneous, be sure to identify it in your final answer.

square root of the quantity x minus 3 end quantity plus 5 equals x

Answers

Answer:

Step-by-step explanation:

[tex]\sqrt{x-3} +5=x\\\sqrt{x-3} =x-5\\squaring ~both~sides\\x-3=x^2-10x+25\\x^2-10x-x+25+3=0\\x^2-11x+28=0\\x^2-7x-4x+28=0\\x(x-7)-4(x-7)=0\\(x-7)(x-4)=0\\x=7,4[/tex]

put x=7 in the given equation

[tex]\sqrt{7-3} +5=7\\\sqrt{4} +5=7\\2+5=7\\7=7[/tex]

which is true .

∴ x=7 is a solution of the given eq.

now put x=4 in the given eq.

[tex]\sqrt{4-3} +5=7\\1+5=7\\6=7\\[/tex]

which is not true.

∴x=4 is an extraneous solution.

. Use the quadratic formula to solve each quadratic real equation. Round
your answers to two decimal places. If there is no real solution, say so.
a) x^2 - 5x + 11 = 0
b) -2x^2 - 7x + 15 = 0
c) 4x^2 - 44x + 121 = 0​

Answers

Answer:

A. No real solution

B. 5 and -1.5

C. 5.5

Step-by-step explanation:

The quadratic formula is:

[tex]\begin{array}{*{20}c} {\frac{{ - b \pm \sqrt {b^2 - 4ac} }}{{2a}}} \end{array}[/tex], with a being the x² term, b being the x term, and c being the constant.

Let's solve for a.

[tex]\begin{array}{*{20}c} {\frac{{ 5 \pm \sqrt {5^2 - 4\cdot1\cdot11} }}{{2\cdot1}}} \end{array}[/tex]

[tex]\begin{array}{*{20}c} {\frac{{ 5 \pm \sqrt {25 - 44} }}{{2}}} \end{array}[/tex]

[tex]\begin{array}{*{20}c} {\frac{{ 5 \pm \sqrt {-19} }}{{2}}} \end{array}[/tex]

We can't take the square root of a negative number, so A has no real solution.

Let's do B now.

[tex]\begin{array}{*{20}c} {\frac{{ 7 \pm \sqrt {7^2 - 4\cdot-2\cdot15} }}{{2\cdot-2}}} \end{array}[/tex]

[tex]\begin{array}{*{20}c} {\frac{{ 7 \pm \sqrt {49 + 120} }}{{-4}}} \end{array}[/tex]

[tex]\begin{array}{*{20}c} {\frac{{ 7 \pm \sqrt {169} }}{{-4}}} \end{array}[/tex]

[tex]\begin{array}{*{20}c} {\frac{{ 7 \pm 13 }}{{-4}}} \end{array}[/tex]

[tex]\frac{7+13}{4} = 5\\\frac{7-13}{4}=-1.5[/tex]

So B has two solutions of 5 and -1.5.

Now to C!

[tex]\begin{array}{*{20}c} {\frac{{ -(-44) \pm \sqrt {-44^2 - 4\cdot4\cdot121} }}{{2\cdot4}}} \end{array}[/tex]

[tex]\begin{array}{*{20}c} {\frac{{ 44 \pm \sqrt {1936 - 1936} }}{{8}}} \end{array}[/tex]

[tex]\begin{array}{*{20}c} {\frac{{ 44 \pm 0}}{{8}}} \end{array}[/tex]

[tex]\frac{44}{8} = 5.5[/tex]

So c has one solution: 5.5

Hope this helped (and I'm sorry I'm late!)

What is m Round the answer to the nearest whole number.
O 30°
O 35°
O 55°
O 60°

Answers

Answer:

30

Step-by-step explanation:

fufyfuf7fjcjcufuy7fufucyyxyvkbuvufudydy shut up

help me im dangered plzzzzzzzzzzzzzzzzzzzz

Answers

Answer:

A

Step-by-step explanation:

Hi!

An exponent is the same thing as just multiplying the expression by itself the number of times the exponent says. So we need to multiply 1/3 by itself three times.

1/3 * 1/3 * 1/3 = 1/27

the answer to your question is A

how to find the theta with side lengths of a triangle

Answers

Step-by-step explanation:

Hello, there!!!

I hope you mean the question is like the above problem in picture.

so, let's simply work with it.

here, we may use cosine rule,

so, according to cosine rule,

[tex] {c}^{2} = {a}^{2} + {b}^{2} - 2ab.cosc[/tex]

so, just put value of formulae here,

we get;

5^2 = 3^2 + 4^2 - (2×3×4) . cos thita

or, 25 = 9 + 16 -24 cos thita.

or, 24 cos thita = 0

or, cos thita = 0/25

or, cos thita = 0

now, taking cos to right side we get,

[tex] {cos}^{ - 1 } (0)[/tex]

now, after typing cos ^-1 (0) we get angle as 90°.

(note: in step {cos thita = 0} you couold directly write like; cos thita = cos 90°. and cos would be cancelled in it as cos 90°=0. but it is only applied in particular angle like 0°,30°,60°,..... which are identified or if you don't know you must use the method above using calculator and remember to put inverse {cos^-1}).

so, In this way we find angle.

I hope it helps....

help help help me plZZZZZ ill give you brainly ;DDD

Answers

Answer:

the answer is 60.7

Step-by-step explanation:

60 to has a between numbers like given in the picture

so as number line it's

60.1 . 60.2 60.3 60.4 60.5 60.6 60.7 60.8 and continue

if u get any 3 digit number like 600 to 650 in number line

u do it like it the same 600.1 600.2.... and go on

Answer:

63½ or 63.5

Step-by-step explanation:

65-60=5

10points=5

1point=?

1×5/10= ½

that means the sequence continues after adding ½ i.e

60..60½...61...61½...62...62½...63...63½...64..64½...65

you have been asked the 8th number which is 63½

1. Find the greatest common divisor of the term 144x3y2and 81xy4​

Answers

Answer:

[tex]1296x^3y^4[/tex]

Step-by-step explanation:

Given the terms:

[tex]144x^3y^2[/tex]

and [tex]81xy^4[/tex]

To find:

Greatest Common Divisor of the two terms or Least Common Multiple (LCM) of two numbers = ?

Solution:

First of all, let us find the HCF (Highest Common Factor) for both the terms.

i.e. the terms which are common to both.

Let us factorize them.

[tex]144x^3y^2 = \underline{3 \times 3} \times 16\times \underline x \times x^{2}\times \underline{y^{2} }[/tex]

[tex]81xy^4= \underline {3\times 3}\times 9 \times \underline{x} \times \underline{y^2}\times y^2[/tex]

Common terms are underlined.

So, HCF of the terms = [tex]9xy^2[/tex]

Now, we know the property that product of two numbers is equal to the product of the numbers themselves.

HCF [tex]\times[/tex] LCM = [tex]144x^3y^2[/tex] [tex]\times[/tex] [tex]81xy^4[/tex]

[tex]LCM = \dfrac{144x^3y^2 \times 81xy^4}{9xy^2}\\\Rightarrow LCM = 144x^3y^2 \times 9x^{1-1}y^{4-2}\\\Rightarrow LCM = 144x^3y^2 \times 9x^{0}y^{2}\\\Rightarrow LCM = \bold{1296x^3y^4 }[/tex]

what is the answer for 6x-4=-26+5x

Answers

Answer: x=-22

Step-by-step explanation:

    6x-4=-26+5x

6x-4-5x=-26+5x-5x ⇔ subtraction property of equality

       x-4=-26

  x-4+4=-26+4 ⇔ addition property of equality

         x=-22

Answer:

x = - 22

Step-by-step explanation:

6x - 4 = - 26 + 5x

First of all group like terms

Send the constants to the right side of the equation and those with variables to the left

That's

6x - 5x = 4 - 26

Simplify

We have the final answer as

x = - 22

Hope this helps you

+
If the
sides of a triangles are
6, 8 and n. how
many integer values of n
could be the
measure of the
third side of the triangle?

Answers

Answer:

11

Step-by-step explanation:

The sum of the shortest two sides must be greater than the longest side.

If n is the longest side:

6 + 8 > n

14 > n

If 8 is the longest side:

6 + n > 8

n > 2

So n must be an integer greater than 2 and less than 14.

n can be 3, 4, 5, 6, 7, 8, 9, 10, 11, 12, or 13.

There are 11 possible integers.

[tex] \LARGE{ \boxed{ \rm{ \purple{Answer}}}}[/tex]

We know,

Sum of two sides of a triangle > Third side

Then,

⇛ 6 + 8 > n

⇛ 14 > n

Nextly,

Difference of two sides of a triangle < Third side

Then,

⇛ 8 - 6 < n

⇛ 2 < n

Then, Range of third side:

☃️ 2 < n < 14

Possible measures of 3rd sides = 3, 4, 5, 6, 7, 8, 9, 10, 11, 12 or 13.

There are 11 possible values of 3rd side. Out of them, any measure is the length of 3rd side.

━━━━━━━━━━━━━━━━━━━━

Which of the following symbols could correctly finish the statement. Select all that apply. 0___-8 = ≠ > < ≥ ≤

Answers

Answer:

>

Step-by-step explanation:

Even though its 0 its still greater than any negative number.

Answer:

Step-by-step explanation:

Find the missing probability: P(B)=7/20, P(A|B)=1/4, P(A∩B)=?

Answers

Answer:

P(A∩B) = 7/80

P(A∩B) = 0.0875

Step-by-step explanation:

Given

P(B)=7/20

P(A|B)=¼

Required

P(A∩B)=?

The given probability shows conditional probability and the relationship between the given parameters is as follows.

P(A∩B) = P(B) * P(A|B)

Substitute ¼ for P(A|B) and 7/20 for P(B)

The expression

P(A∩B) = P(B) * P(A|B) becomes

P(A∩B) = 7/20 * ¼

P(A∩B) = 7/80

P(A∩B) = 0.0875

Hence, the calculated P(A∩B) is 7/80 or 0.0875

ASAP PLZ ANSWER!!! Can you tell me step by step to this question 8,595 ÷ 24?

Answers

Answer:

358 and remainder of 3

Step-by-step explanation:

1. Divide it like any other problem

24 goes into 85, 3 times with 13 left overBring down the 9 and 24 goes into 139, 5 times with 19 left overThen bring down the 5 and 24 goes inside 195, 8 times with 3 left overSo your remainder would be 3

Hope this helps

find the area of a rhombus with a 120 degree angle and sides 10 cm

Answers

Answer:

              A = 50√3 cm²

Step-by-step explanation:

[tex]A=s^2\cdot \sin\alpha\\\\s=10\\\alpha=120^o\\\\A=10^2\cdot\sin(120^o)=100\sin(180^o-60^o)=100\sin(60^o)=100\cdot\frac{\sqrt3}2=50\sqrt3[/tex]

I also need to know this one pretty soon pleaseee​

Answers

Answer:

128

Step-by-step explanation:

Other Questions
Find an exact value of sin(17pi/12) Write the expression 12-2 in simplest form. I have four brothers"This is what type ofobservation? 1. Why do you see colors when you look at reflected light from a CD or DVD disk, or when you look at a soap bubble or oil film on water?2. What do you think causes the colors on the artwork panels on the side of HLS2 (Health Sciences building) which change with time of day and the angle from which you view them? Abey Kuruvilla, of Parkside Plumbing, uses 1,210 of a certain spare part that costs $26 for each order, with an annual holding cost of $24. a) Calculate the total cost for order sizes of 25, 40, 50, 60, and 100 (round your responses to two decimal places). normal population has a mean of 63 and a standard deviation of 13. You select a random sample of 25. Compute the probability that the sample mean is: (Round your z values to 2 decimal places and final answers to 4 decimal places): Greater than 65. A new fast-food firm predicts that the number of franchises for its products will grow at the rate dn dt = 6 t + 1 where t is the number of years, 0 t 15. John used 1 3/4 kg os salt to melt the ice on the sidewalk. He then used another 3 4/5 kg on the driveway. How much salt did he use in all? PLEASE SHOW YOUR WORK I WILL MARK YOU BRAINIEST AND PLEASE EXPLAIN HOW YOU GOT YOUR WORK. What is systems thinking? identifying the different parts of a system describing how the parts of a system work together analyzing how things outside a system influence the things in that system thinking about how things influence each other within a system PLS HELP ME Define Derived Quantities ? __________________ numbers are used to say the order in which people, things, or events show. what are sex hormones?why are they named so? state their function. Making inferences and drawing conclusions help us with all of the following except __________. A. making connections B. creating new words C. understanding cause and effect D. learning new vocabulary A 28.0 kg child plays on a swing having support ropes that are 2.30 m long. A friend pulls her back until the ropes are 45.0 from the vertical and releases her from rest.A: What is the potential energy for the child just as she is released, compared with the potential energy at the bottom of the swing?B: How fast will she be moving at the bottom of the swing?C: How much work does the tension in the ropes do as the child swings from the initial position to the bottom? o prepare vegetables for finishing by grilling, sauting, pan frying, deep frying, or stewing, you should _______ them to cook them to partial doneness What is the separation in meters between two slits for which 594 nm orange light has its first maximum at an angle of 32.8? In a task assignment situation, in how many different ways can five jobs be assigned to five machines if each job must be assigned to each machine and each machine must be assigned one job? A. 5 B. 1 C. 3,125 D. 120 E. 25 Translate and solve: 54 greater than x is greater than 216 Triangle P Q R is shown. The length of P Q is 17, the length of Q R is 15, and the length of P R is 14. Law of cosines: a2 = b2 + c2 2bccos(A) What is the measure of AngleP to the nearest whole degree? 35 52 57 72 the expansion of islam resulted in european expansion in which of the following ways